Most restrictive placement
Hello! I got this question right, but I am curious, similar to the previous question where we tri...
jbenav246@gmail.com on January 6 at 03:41PM
  • December 2009 LSAT
  • SEC2
  • Q9
1
Reply
Better Approach
May I ask if there is a better way to find the answer choice than to go through each one to deter...
AndreaA on January 6 at 03:32PM
  • December 2009 LSAT
  • SEC2
  • Q8
1
Reply
Diagramming
When I diagrammed this scenario, since F could either be 1 or 7 I made 2 scenarios, one with F be...
rosemarie on January 6 at 03:28PM
  • December 2009 LSAT
  • SEC2
  • Q6
1
Reply
Why isn't D the correct answer?
Why isn't D the correct answer?
drueyoung on January 6 at 03:13PM
  • December 2009 LSAT
  • SEC2
  • Q3
1
Reply
Mistake in Question?
The correct answer according to the video is B, however, the exam reports it as C
Kenny1111 on December 14 at 12:04AM
  • December 2009 LSAT
  • SEC2
  • Q22
1
Reply
Why can't it be E?
Example: L HF IPTS or L HS IPTF
lakshmi on August 17 at 03:08PM
  • December 2009 LSAT
  • SEC2
  • Q4
1
Reply
Why could World History not be the answer?
In the video she says that we know Russian and WH is in and G is out and we don't know anything m...
avramiusmc@gmail.com on April 28, 2023
  • December 2009 LSAT
  • SEC2
  • Q13
1
Reply
Sec 2 Q13
Maybe I missed this but how do you know S9 and S3 cant be together? Can you explain fully how tha...
lxnxrd19 on April 28, 2023
  • December 2009 LSAT
  • SEC2
  • Q13
1
Reply
What is the point of having 2 statistic times?
Hello, What is the LSAT trying to do by having 2 statistic times? Is the primary goal to mess ...
alicat6 on April 28, 2023
  • December 2009 LSAT
  • SEC2
  • Q11
1
Reply
How would we solve this without hypotheticals?
Using just set up.
Thalia on April 28, 2023
  • December 2009 LSAT
  • SEC2
  • Q23
1
Reply
Must be false
Just as a reminder, can we assume that must be false = never be true?
jbenav246@gmail.com on April 28, 2023
  • December 2009 LSAT
  • SEC2
  • Q23
1
Reply
Scenarios
With the highly restricted variable M/T and V/L, why did Naz in her set-up not do any scenarios? ...
bb042745 on April 28, 2023
  • December 2009 LSAT
  • SEC2
  • Q17
1
Reply
Is there a faster way to find the answer
Rather than going through all the answer choices is there a faster way to find the answer ?
Nativeguy on April 25, 2023
  • December 2009 LSAT
  • SEC2
  • Q19
1
Reply
Why couldn't the answer be C? I in number 2
This does determine the the rest of the board and does not break any rule.
Nativeguy on April 25, 2023
  • December 2009 LSAT
  • SEC2
  • Q8
2
Replies
Please dumb it down - Why not A
If H is 2nd and F is 7th M H G K I L F - works
Lena2 on March 1, 2023
  • December 2009 LSAT
  • SEC2
  • Q10
2
Replies
Please explain why B isn't correct?
I understand why D also works, but for rule substitution questions- arent we just looking for a c...
Eaten on November 9, 2022
  • December 2009 LSAT
  • SEC2
  • Q10
1
Reply
What is the "most restrictive" rule?
How do we determine which block is more restrictive (IL or GK)? Technically, wouldn't GK be more ...
skraps17 on June 22, 2022
  • December 2009 LSAT
  • SEC2
  • Q8
1
Reply
Why is this not a guru game?
I've watched the video explanation for this game, but everytime I see this game I'm tempted to se...
Karen-Norris on February 1, 2022
  • December 2009 LSAT
  • SEC2
  • Q19
3
Replies
Every single one is correct
Hoping someone can help me here. all the answers are correct T comes first M comes 4th. What exa...
lilshayla11@yahoo.com on January 19, 2022
  • December 2009 LSAT
  • SEC2
  • Q18
1
Reply
Stumped
I was able to narrow my answers down to J R and M but since the rules says J/R and J/M I couldn't...
tomgbean on June 21, 2021
  • December 2009 LSAT
  • SEC2
  • Q15
4
Replies